Old Q&A – Male Reproductive System

A 56-year-old man with coronary artery disease comes to the clinic for follow up of a prostate specific antigen test (PSA). He was seen 3 weeks ago and a screening PSA level was drawn. The test result came back at 9.4 ng/mL and the patient returns to discuss this result. His medications are atenolol, pravastatin, enalapril, and aspirin. He denies any symptoms of urinary retention, hesitancy, pain on urination, and post-void dribbling. The most appropriate next step in management is to

  A. arrange for a prostate biopsy
  B. follow up with the patient in 6 months and obtain a second PSA level
  C. inform the patient that he has benign prostatic hypertrophy
  D. inform the patient that he has prostate cancer
  E. reassure him that he does not have prostate cancer
Explanation:

The correct answer is A. Men have between a 15-20% lifetime risk of developing prostate cancer. The PSA test was introduced in the late 1980s and quantifies a glycoprotein produced by the prostate that spills over into the blood. Although current guidelines differ by society, clinical practice and standard of care is to test high risk men over the age of 40 years for PSA levels. Any level greater than 4.0 ng/mL requires a prostate biopsy. The positive and negative predictive values of the test vary tremendously with patient population, but roughly 30% of patients with elevated PSA levels will have prostate cancer.

Although some physicians follow marginal elevations in PSA levels for a few months (choice B), this patient has a nearly 3-fold increase above that required for further workup (4.0ng/mL).

Just as the PSA test has poor sensitivity, its specificity (false positives) is also variable, ranging from 80-97%. For this reason, elevations may not indicate the presence of cancer, but perhaps are due to benign prostatic hypertrophy (choice C). However, until the biopsy is performed, a determination as to the cause of the PSA elevation is not possible (choice D). As stated above, roughly 30% of patients with elevated PSA levels will have prostate cancer (the probability of prostate cancer given an elevated PSA is 30%).

Since the incidence of false-negative results is approximately 30-50% (sensitivity), there is no possible way to determine that this patient is cancer free unless biopsies are taken (choice E).

 

A 65-year-old man comes to the clinic for a periodic health maintenance examination. He claims to be healthy and has not seen a physician in almost 10 years. His only complaint is of recent onset lower back pain. He has no significant past medical history, has never had surgery, does not take any medications, and has no known drug allergies. His family history reveals maternal death at age 75 of a heart attack and paternal death at age 54 of colon cancer. He feels generally well and is very active. He is happily married and plays tennis 3 times a week. His temperature is 36.9 C (98.4 F), blood pressure is 142/78 mm Hg, pulse is 70/min, and respirations are 22/min. Physical examination is unremarkable. Laboratory values are within normal limits except that his PSA, which he pressured you to order, is 45. You refer him to a urologist who performs a transrectal ultrasound-guided biopsy which reveals an adenocarcinoma Gleason 4+4=8 on both sides of the prostate. A bone scan shows areas of increased uptake diffusely, especially in the lumbar spine region, which is suspicious for metastatic disease. The most appropriate first-line treatment for this patient is

  A. brachytherapy
  B. chemotherapy
  C. external beam radiation
  D. hormone therapy
  E. prostatectomy
Explanation:

The correct answer is D. This patient has metastatic prostate cancer, a clinical dilemma. The current mainstay of treatment for metastatic prostate cancer is androgen deprivation therapy using a leuteinizing hormone-releasing hormone (LHRH) agonist such as leuprolide or goserelin or by performing a bilateral orchiectomy. Prostate cancer depends on androgen stimulation for neoplastic activity. LHRH agonists cut off testicular testosterone secretion and thereby, decrease the activity of metastatic lesions. LHRH agonists must initially be given with an androgen blocker such as flutamide due to the initial flare reaction, which causes an increase in serum testosterone and may increase bony pain. The dilemma of metastatic prostate cancer is that the disease eventually becomes hormone refractory, meaning it no longer is suppressed by androgen deprivation. The disease typically becomes hormone refractory within 3 years of the onset of treatment. Therefore, many patients with metastatic disease are not started on hormone therapy right away. Many clinicians wait until the patient becomes symptomatic with bone pain, before initiating therapy.

Brachytherapy (choice A), or interstitial radiation, is used for organ-confined disease. Iodine125 or palladium103 seeds are implanted into the prostate controlling local disease. The results of brachytherapy have been comparable to surgery in terms of disease-free survival.

Chemotherapy (choice B) plays a very limited role in the treatment of prostate cancer. It is occasionally used for patients with hormone refractory disease. However, it has not been shown to have any survival benefit.

External beam radiation (choice C) is used as a treatment for localized prostate cancer, especially for patients who are poor surgical candidates. It is also used as adjuvant therapy for positive surgical margins and recurrent localized disease after prostatectomy.

Prostatectomy (choice E) is an ideal treatment for patients who have organ-confined disease. Organ-confined disease is suggested by a negative metastatic workup and negative pelvic lymph node dissection. Patients with lower PSA and lower Gleason scores are more likely to have organ-confined disease and are therefore, better surgical candidates.

 

A couple who you have been treating for many years for various “colds and viruses” comes to the office because they have been unsuccessfully trying to conceive for the past 3 years. They say that they are enjoying the “act of trying” but are getting a bit concerned that there is something “wrong”. The wife is 32 years old, has never had a sexually transmitted disease and has never been pregnant before. She has had regular menstrual periods since she was 14 years old and usually has cramping and breast tenderness a few days before menses. The husband is 36 years old and denies any sexually transmitted diseases. He is an avid cyclist and goes on 10-mile rides each day. Neither of them takes any medications. You perform a complete physical examination on both of the patients and find no abnormalities. During the pelvic examination, you obtain a Pap smear, gonorrhea and chlamydia cultures. You order thyroid function tests, prolactin levels, and a mid luteal serum progesterone level in the wife and advise her to record her basal body temperature. The couple returns to the office 1 month later to go over the test results. All of the studies that you ordered were normal, and the results of the basal body temperature show a 0.6% temperature rise at day 14 that remains elevated until 13 days later. The temperature drops and menses occurs 24 hours later. The most appropriate next step is to

  A. advise him to stop bicycling so often
  B. determine his testosterone concentration
  C. inform them that she is not ovulating
  D. obtain a semen sample for analysis
  E. schedule a hysteroscopy
Explanation:

The correct answer is D. Infertility is usually defined as the failure to conceive after a year of unprotected intercourse and it affects up to 15% of reproductive-aged couples. 60% of the time there is a female factor such as ovulation disorders or anatomical defects in the genital tract and the other 40% of the time it is due to male disorders of spermatogenesis. The initial work-up of an infertile couple usually includes a complete history and physical examination, a basal body temperature chart, laboratory studies, and semen analysis obtained by masturbation. The semen analysis evaluates the sperm count, volume, viscosity, motility, and differential. A hysterosalpingogram, which is an x-ray of the female genital tract after an opaque dye is injected into the uterine cavity, is useful in evaluating the anatomy and is sometimes included in the initial work-up. However, many OB-GYNs will order this study only after the results of the previously mentioned studies are normal.

The most appropriate next step is to obtain a semen sample for analysis, not to advise him to stop bicycling so often (choice A). While some believe that the pressure and heat generated by sitting on a bicycle seat can affect sperm count, you must first order a semen analysis to determine if the infertility is due to a male factor.

Determining his testosterone concentration (choice B) is not a typical part of the evaluation of infertility. Semen analysis is important to determine the sperm count, volume, viscosity, motility, and differential.

It is incorrect to inform them that she is not ovulating (choice C) because she has regular menstrual cycles with menstrual symptoms and the results of the basal body temperature recording are completely normal and indicate that she is most likely ovulating. Also, the mid luteal progesterone is normal.

It is inappropriate to schedule a hysteroscopy (choice E) at this time. A hysteroscopy allows the physician to directly evaluate the endometrial cavity through an endoscope and to possibly biopsy or remove any lesions that are present. A semen analysis should be performed before an invasive procedure, such as this, is ordered.

 

You are performing a discharge examination on a 1-day old healthy newborn. He was born by a normal spontaneous vaginal delivery at 39 weeks gestation. During the routine physical examination you identify the right testicle, but are unable to palpate the left testicle. Palpation of the left inguinal canal does not reveal a mass. The rest of the examination is normal. The most appropriate management at this time is to

  A. do a CT scan of the pelvis to search for an undescended testis
  B. follow up as an outpatient and if no testis are present at one year of age, then refer to urology for possible orchiopexy
  C. follow up as an outpatient and if no testis is present at ten years of age, refer to urology for possible orchiopexy
  D. reassure the parents that this is common and no treatment or follow-up is necessary
  E. refer to urology for orchipexy prior to discharge
Explanation:

The correct answer is B. This patient has an absent testis in the left hemiscrotum. The differential diagnosis for this is cryptorchidism (i.e. undescended testis), retractile testis (i.e. child with a brisk cremasteric reflex causing retraction of testis into inguinal canal), or absent testis (sometimes thought to be due to intrauterine torsion). The most likely diagnosis is cryptorchidism. Cryptorchidism, literally meaning hidden testis, is very common. It is present in approximately 5% of term male babies and up to 30% of premature male babies. The majority descend spontaneously by one year of age decreasing the incidence to only 0.8% that are truly undescended. At that point, where the testis is felt to be undescended, it should be treated with a procedure called orchipexy to bring it down into the scrotum. Not doing this greatly increases the risk of testicular malignancy, torsion, and infertility. The appropriate management is to follow up as an outpatient to evaluate for testicular presence. If no testis are present at one year of age, then referral to urology for possible orchiopexy is necessary.

A CT scan of the pelvis (choice A) particularly as a newborn is unlikely to reveal the location of the testicle and furthermore exposes the child to unnecessary radiation. An ultrasound of the inguinal canal region is either performed prior to discharge or as an outpatient, but this does not usually end up changing management.

Because cryptorchidism increases the risk of testicular malignancy, torsion, and infertility, waiting to treat it, until age 10 (choice C) or not treating at all (choice D) is not acceptable. Furthermore, it is not appropriate to treat too early (i.e. treating as a newborn) (choice E) because of the number of children who have spontaneous resolution.

 

  You are taking care of a 49-year-old woman who was admitted to the hospital because of progressive numbness of the right arm and difficulty in seeing objects in the left visual field. She is known to be HIV positive, but has not consistently taken medications in the past. On examination she is healthy appearing, has a right homonymous hemianopia, and decreased sensory perception in her left upper extremity and face. Her CD4 count is 60 cells/mm3 and her MRI is consistent with a demyelinating lesion of the left parietooccipital area. CSF PCR for the JC virus is positive. The most appropriate treatment is

  A. amphotericin B
  B. cranial radiation
  C. highly active antiretroviral therapy (HAART)
  D. intravenous acyclovir
  E. intravenous ceftriaxone
Explanation:

The correct answer is C. This patient has progressive multifocal leukoencephalopathy. It is caused by the JC virus, which is a double stranded DNA virus. The prognosis is poor, but HAART has been known to be effective in improving survival. The JC virus is ubiquitous and may be transmitted through respiratory secretions.

Amphotericin B (choice A) is used to treat fungal infections, it is not used to treat progressive multifocal leukoencephalopathy or the JC virus.

Cranial radiation (choice B) is used to treat malignancies. This patient has a demyelinating lesion of the brain and a positive CSF PCR for the JC virus, not a malignancy.

Intravenous acyclovir (choice D) is not effective against JC virus, but is used to treat the herpes simplex virus encephalitis.

Intravenous ceftriaxone (choice E) is used to treat bacterial meningitis. Bacterial meningitis typically presents with a fever, nuchal rigidity, and a headache. This patient’s presentation and findings are inconsistent with bacterial meningitis.

 

A 19-year-old man comes to the clinic with a gradually worsening scrotal pain for the past week. He has no significant past medical history. He says he is sexually active with 2 partners and uses condoms “occasionally”. General physical examination is normal. Examination of the genitalia reveals a very tender left epididymis. The testes are normal. There is a whitish discharge from the penile meatus. Transillumination of the scrotum demonstrates no evidence for a hydrocele. To exclude testicular torsion, ultrasonography of the testes is performed. The right testicle and epididymis are normal (not shown). The left testicle is normal. The findings from the right epididymis are shown (gray-scale and color Doppler). The intervention most likely to have prevented this condition is

  A. evaluation for undescended testis
  B. prophylactic antibiotics
  C. regular medical check-ups
  D. safe sex counseling
  E. support underwear
Explanation:

The correct answer is D. Epididymitis has progressed to an epididymal abscess in this patient. The ultrasound reveals a hypoechoic collection in the epididymis and on Doppler images the mass has no vascular flow. Epididymitis is caused by retrograde infection from any sexually-transmitted organism secondary to sexual intercourse. It is rarely spread hematologically except in some immunocompromised patients.

The testicles are not undescended (choice A) given the scrotal exam. Undescended testes increases the risk for testicular cancer, which is not what this patient has.

Prophylactic antibiotics (choice B) are not indicated in the general population without a history of recurrent epididymitis.

Regular medical check-ups (choice C) would not have prevented this sporadic disease occurrence. They are important, however, for general health maintenance.

Support underwear (choice E) has no relation to epididymal or testicular infections.

 

A 54-year-old man comes to your office for his yearly physical examination. You have been his primary care physician for the last 18 years. He is in good health without any chronic medical conditions. His social history includes a 45-pack-year history of tobacco use and 20 years of working in a textile factory. His father has prostate cancer and diabetes. His mother, brother, and sister are all healthy. Review of his urologic history is noncontributory. In the past, his rectal examination and prostate specific antigen (PSA) have always been normal. Examination of his genitourinary system today reveals a circumcised penis without discharge or lesions, and testicles that are descended and normal bilaterally. On digital rectal examination you palpate a hard nodule over the left apex of the prostate. Stool is guaiac positive. PSA is 7.4 ng/mL. The findings that indicate the need for this patient to undergo a prostate biopsy is/are

  A. elevated PSA and/or nodule on prostate
  B. elevated PSA only
  C. exposure to risk factors at work
  D. family history of prostate cancer
  E. family history of prostate cancer and elevated PSA
  F. guaiac-positive stool
  G. history of smoking
  H. history of smoking and work exposure
  I. nodule on prostate only
  J. nodule on prostate and family history of prostate cancer
Explanation:

The correct answer is A. The screening tests for prostate cancer are digital rectal exam and serum PSA levels. If either one of these is abnormal then the patient needs to undergo transrectal ultrasound with prostate biopsy. This procedure can be done as an outpatient without anesthesia. Utilizing transrectal ultrasound allows for visualization of the prostate at the time of biopsy so that each specimen is from a different anatomic location within the prostate. In general, normal PSA levels are <4 ng/mL. Some investigators believe in “age-adjusted PSA.” In that case, the upper limits of normal PSA for men ages 40-49 is 2.5 ng/mL, ages 50-59 is 3.5 ng/mL, ages 60-69 is 4.5 ng/mL, and ages 70-79 is 6.5 ng/mL. Under either system, this patient’s PSA of 7.4 ng/mL is abnormal and warrants further investigation. The fact that this patient has both an abnormal rectal exam and an elevated PSA are even stronger indications for prostate biopsy. The urologic literature is filled with a variety of blood tests/imaging studies that may be performed in an attempt to limit the number of men undergoing transrectal ultrasound and prostate biopsy. To date none of these have been uniformly accepted. This leaves only the digital rectal exam and serum PSA levels as the initial screening tools for prostate cancer.

Elevated PSA (choice B) and nodule of the prostate (choice I ) are both indications for prostate biopsy. As stated, if either is abnormal then the patient is a candidate for biopsy. However, (choice A) is the correct answer because it acknowledges that both are abnormal in this patient.

This patient’s history of tobacco use (choices G and H) and exposure while working in a textile factory (choice C) are risk factors for development of transitional cell carcinoma of the urinary tract (ureter/bladder). Neither of these are indications for prostate biopsy.

The fact that this patient’s father has prostate cancer (choices D, E, J) is concerning. This patient is eligible for earlier and perhaps more frequent screening tests. However, a family history of prostate cancer is not an indication to perform prostate biopsy. In choice J, the presence of a nodule on DRE is indication for biopsy; however, the family history is not an indication.

Guaiac-positive stool (choice F) is an important finding on this patient’s physical exam. It will warrant further investigation but is not an indication for prostate biopsy.

 

A 25-year-old man is admitted to the hospital after sustaining head injuries in a motor vehicle accident. On his 2nd day in the hospital, he shows you a sore on his penis that he developed a few days ago. He proudly admits to numerous sexual encounters in the past 5 years, and tells you that he has been tested for HIV every 6 months, and that the last negative test only was about 3 months ago. He is otherwise generally healthy, and does not take any medications on a regular basis. He denies any penile discharge in the past or present, and no history of other sexually transmitted diseases. On physical examination, there is painful lymphadenopathy of the left groin region. On the distal penis, there are 2 tender, ragged ulcers that appear punched out with surrounding hyperemia. The base of the ulcers are covered with a purulent, dirty exudate, which bleeds easily during the examination. This patient most likely has

  A. chancroid and should be treated with azithromycin 1g orally in a single dose, while cultures and tests for other sexual transmitted diseases are performed
  B. gonorrhea dermatitis and he should receive therapy for both gonorrhea and chlamydia, while cultures and tests for other sexual transmitted diseases are performed
  C. granuloma inguinale and should be treated with trimethoprim-sulfamethoxazole, while cultures and tests for other sexual transmitted diseases are performed
  D. lymphogranuloma venereum and should be treated with doxycycline, while cultures and tests for other sexual transmitted diseases are performed
  E. a primary syphilitic chancre and serum RPR should be checked to confirm the diagnosis, while cultures and tests for other sexual transmitted diseases are performed
Explanation:

The correct answer is A. Chancroid is correct because it usually presents as an inflammatory papule that ruptures early with the formation of a ragged ulcer that lacks the induration of a chancre. The ulcers have undermined irregular edges surrounded by mild hyperemia, and the base is usually covered with purulent, dirty exudate. This is an infectious, contagious, ulcerative, sexually transmitted disease caused by the Gram-negative bacillus Haemophilus ducreyi. It is characterized by 1 or more deep or superficial tender ulcers on the genitalia and painful unilateral inguinal adenitis. However, the diagnosis of chancroid does not rule out syphilis and the subsequent development of syphilis should be anticipated since the incubation time for a chancre is longer than that of chancroid.

Gonococcal dermatitis (choice B) is incorrect because it is a rare infection that occurs mostly as erosions on the median raphe without urethritis. Grouped pustules on an erythematous base is the usual presentation.

Granuloma inguinale (choice C) is incorrect because it is a mildly contagious, chronic, granulomatous, locally destructive disease characterized by progressive indolent, serpiginous ulcerations of the groins, pubis, genitalia, and anus. The disease begins as single or multiple subcutaneous nodules, which erode through the skin to produce clean, sharply defined lesions, which are usually painless. The lesions typically demonstrate hypertrophic, vegetative granulation tissue which is soft, has a beefy-red appearance, and bleeds readily. The regional lymph nodes are usually not enlarged. This is caused by Calymmatobacterium granulomatis.

Lymphogranuloma venereum (choice D) is incorrect, because it is a sexually transmitted disease characterized by suppurative inguinal adenitis with matted lymph nodes, inguinal bubo with secondary ulceration, and constitutional symptoms. The primary lesion consists of herpetiform vesicle or erosion develops on the glans penis followed by bilateral lymphadenopathy. It is caused by Chlamydia trachomatis, serotypes L1, L2 and L3.

Primary syphilitic chancre (choice E) is incorrect, because it typically presents as a crusted superficial erosion that becomes a round or oval, indurated, slightly elevated papule, with an eroded, but not ulcerated surface that exudes a serous fluid. The lesion is usually painless. The regional lymph nodes on one or both sides are usually enlarged, firm, nontender and do not suppurate. With this said, when a patient presents with a penile ulcer, it is wise to obtain a serum RPR since patients can often times have more than one sexually transmitted disease.

 

A 27-year-old man comes to the emergency department because of an “exquisitely painful” scrotum. He says that he was walking to lunch with friends when the pain hit him “like a thunderclap.” He says that he has a steady girlfriend and that they have an “active sex life.” He is “very healthy” and has never experienced pain like this before. He regularly checks himself “there” after that young comedian underwent testicular surgery on television. His temperature is 37 C (98.6 F), blood pressure is 130/85 mm Hg, pulse is 86/min, and respirations are 19/min. Physical examination shows severe scrotal tenderness that is not relieved when the scrotum is elevated. The right testes is high in the scrotum and riding in a horizontal position. The cord above the testes is not tender. A urinalysis shows:

Color straw/light Microscopic
Specific gravity 1.020 WBC 4/hpf
pH 5.8 Glucose absent
Protein absent Bacteria absent

The most appropriate next step is to

  A. administer ciprofloxacin, intravenously
  B. apply ice packs and observe in the emergency department
  C. measure serum HCG and AFP
  D. perform a trans-scrotal testicular biopsy
  E. request a urology consultation, STAT
Explanation:

The correct answer is E. This patient has the classic presentation of testicular torsion, which is a surgical emergency, and therefore requires an immediate urologic consultation. He requires surgical intervention to reverse the spermatic cord torsion and restore blood flow.

Antibiotics, such a ciprofloxacin (choice A) are necessary to treat epididymitis, which typically presents with scrotal pain, fever, pyuria, a tender cord, and a normal positioned testes. Elevation of the testes may somewhat relieve the pain. Ice packs, bed rest, antiinflammatory agents, and scrotal support are typically used in conjunction with the antibiotics.

Applying ice packs and observing him in the emergency department (choice B) is inappropriate management of testicular torsion, which requires immediate urologic consultation.

Measuring serum HCG and AFP (choice C) is part of the evaluation for testicular cancer, which typically presents as a painless mass. If tenderness is present, it is often dull and aching, rarely acute and “exquisitely painful.”

Performing a trans-scrotal testicular biopsy (choice D) is not useful in testicular torsion, which is a surgical emergency, and therefore requires an immediate urologic consultation. A trans-scrotal testicular biopsy is usually avoided, even if a testicular malignancy is suspected, to prevent potential tumor contamination of the lymphatics.

 

A previously healthy 15-year-old boy is brought to your emergency department with a 3-hour history of right testicular pain. He states that the pain began after football practice this afternoon. He does not remember any trauma to the area during practice. He appears to be in a significant amount of pain. His blood pressure is 128/80 mm Hg and his pulse is 110/min. Physical examination shows an erythematous, swollen right scrotum with significant tenderness to palpation on that side. You also note that the cremasteric reflex is absent on the right side. A urinalysis was sent from triage and is negative. The most appropriate next step is a

  A. CT of the abdomen
  B. surgical consultation
  C. urine for gonorrhea and chlamydia
  D. testicular biopsy
  E. voiding cystourethrogram
Explanation:

The correct answer is B. The patient in this vignette has torsion of the right testicle. In any patient with testicular swelling and pain, surgical consultation should be sought as soon as possible because delay in the diagnosis of testicular torsion can result in loss of that testis due to absent blood flow. If the diagnosis is delayed >6 hrs without intervention, there is a significant risk of necrosis of that testis. An ultrasound with color Doppler will evaluate the blood flow to the affected testis and also assess the morphology of the testicle. However, it often delays the management of the condition, which could have disastrous effects.

A CT scan of the abdomen (choice A) will not be helpful in assessing this patient’s scrotum.

Sending urine for gonorrhea and chlamydia (choice C) is an important consideration in a sexually active adolescent who presents with symptoms of epididymitis. Epididymitis is one of the main causes of acute painful scrotal swelling in sexually active young men. Usually the urinalysis will reveal pyuria. This is not a diagnostic test that should supercede the ultrasound with the Doppler, due to the importance of early identification of reduced/absent blood flow.

Testicular biopsy (choice D) is not part of the evaluation of testicular torsion.

A voiding cystourethrogram (choice E) is a procedure used to diagnose vesicoureteral reflux. This test is used as part of the work-up of a child with a urinary tract infection.

 

 

A 55-year-old man comes to the emergency department with pain on urination, fever and chills. He also complains of perineal and suprapubic tenderness as well as dysuria and hesitancy. His allergies include codeine, sulfonamides, and quinidine. Temperature is 38.5 C (101.3 F), blood pressure is 132/90 mm Hg, pulse is 88/min, and respirations are 18/min. Abdominal examination is remarkable for suprapubic tenderness. Digital rectal examination demonstrates a swollen, boggy, and exquisitely painful prostate gland. Laboratory studies show a leukocyte count of 11,500/mm3, creatinine of 0.9 mg/dL, and blood urea nitrogen of 16 mg/dL. A urinalysis shows too numerous to count white blood cells and Gram-negative rods. The most appropriate treatment for this patient is

  A. amoxicillin/clavulanate 875 mg by mouth twice daily for 14 days
  B. ceftriaxone 1 gram intravenously daily for 5 days
  C. ciprofloxacin 500 mg by mouth twice daily for 14 days
  D. clindamycin 300 mg 4 times daily for 10 days
  E. trimethoprim-sulfamethoxazole 1 double strength tablet twice daily for 14 days
Explanation:

The correct answer is C. This patient is presenting with fever, dysuria, and a very tender prostate on examination. These findings are classic for acute prostatitis. The question posed is basically the appropriate treatment for this condition, which is either a fluoroquinolone, such as ciprofloxacin, or a sulfa-based drug such as trimethoprim-sulfamethoxazole (to which he is allergic) (choice E). Ciprofloxacin clearly makes the best choice.

The other antibiotics mentioned in the remaining options (choices A, B, and D) simply are not first line, or appropriate agents for this disease and are thus incorrect choices.

 

A 19-year-old man who is in the hospital because of an asthma exacerbation, has a painful sore on his penis. He tells you that 4 days prior to admission, he had unprotected sexual intercourse with a new partner. Yesterday, he began developing “painful sores” over the distal aspect of his penis. He also complains of dysuria, but denies fevers, chills, meatal discharge, or any previously similar episodes. Three months ago he had an HIV test which was negative. He has bilateral inguinal adenopathy, which is firm and tender to palpation. There is no discharge elicited from the meatus. Dispersed on the penile shaft are multiple small tender vesicles on an erythematous base. Rectal examination shows normal sphincter tone with a firm, appropriately sized, non-tender prostate. Urine dipstick is negative for any sign of infection. You send off a culture from one of the lesions. The next best step in the management of this patient is to

  A. give 1 intramuscular injection of benzathine penicillin G
  B. prescribe ceftriaxone 250 mg intramuscularly, a single dose
  C. prescribe oral acyclovir
  D. prescribe topical acyclovir
  E. prescribe azithromycin 1gm ; orally a single dose
  F. repeat the HIV test
  G. wait for culture results to return
Explanation:

The correct answer is C. This patient has a classic case of primary genital herpes. Typically, this presents as penile lesions of grouped vesicles on an erythematous base that do not follow a neural distribution. The lesions are tender to touch and the associated adenopathy is bilateral, mildly tender, non-fixed, and slightly firm. The primary episode is more severe, than recurrent attacks and the incubation period is 2-10 days. The herpes simplex virus is a double-stranded DNA virus capable of causing persistent and latent infections. Most genital herpes are caused by type 2 virus, however, up to 25% of genital herpes may be caused by type 1 virus. Partners of infected patients are at risk of transmission, even when the virus is asymptomatic. Acyclovir is the only drug that has shown efficacy in the treatment of the signs and symptoms of genital herpes, however, there is no known cure. The medication works by acting as an inhibitor of viral DNA polymerase and acts as a chain terminator. It treats the symptoms by decreasing the duration of viral shedding, the time of crusting of the lesions, and the time for healing of the lesions. Topical acyclovir (choice D) is much less effective than oral or IV therapy and is therefore discouraged.

Benzathine penicillin G(choice A) is used for the treatment of primary syphilis. Syphilis is caused by the spirochete Treponema pallidum and the primary disease presents as a painless, firm, indurated chancre. Adenopathy may be tender or non-tender and is typically firm and “rubbery”.

IM ceftriaxone (choice B) is used in the treatment of chancroid. The causative agent in this disease is Haemophilus ducreyi. The ulcer associated with chancroid is deep with an undermined border and a friable base that bleeds easily. The adenopathy in painful and with chronic infection may cause lymphatic obstruction. One gram of azithromycin (choice E) may also be utilized in the treatment of chancroid.

Repeating the HIV test (choice F) in this patient with high-risk behavior is appropriate. However, it may be performed after treatment of his herpes is initiated. This test should not delay the necessary immediate intervention.

The most sensitive technique for diagnosing herpes is to isolate the virus in a culture. However, results take 5 days and therapy should not be withheld if clinical suspicion for herpes is high (choice G).

 

You are seeing a 63–year-old man on rounds in the medical intensive care unit who was admitted with sepsis related to an infected diabetic foot ulcer. During his admission, he has had multiple complications including respiratory failure, a large perioperative myocardial infarction during a left below the knee, amputation, and atrial fibrillation, which resulted in an embolic stroke. He has been intubated and ventilator dependent since admission. Over the past 3 days his condition has been slowly improving and he is starting to regain consciousness. He now indicates that he is having pain in his scrotum. His temperature is 37.0 C (98.6 F), blood pressure is 112/76 mm Hg, pulse is 92/min, respirations are 22/min (on ventilator). His jugular veins are distended, and his heart is irregularly irregular with an S3 gallop. His lungs have course breath sounds bilaterally, abdomen is mildly distended, and his scrotum is markedly and symmetrically enlarged to approximately four times normal size. There is 4+ pitting edema in the lower extremities bilaterally. An ultrasound of the scrotum is performed which shows normal testes and diffuse thickening of the scrotal skin and a small to moderate sized hydrocele on the left and a small hydrocele on the right. The most appropriate course of treatment for his scrotal pain is

  A. ciprofloxacin 500 mg via nasogastric tube twice daily for 14 days
  B. diuresis as tolerated by his volume status
  C. no specific treatment would help
  D. percutaneous aspiration of the hydroceles
  E. percutaneous aspiration of the hydroceles followed by placement of drainage tubes bilaterally to prevent reaccumualtion
Explanation:

The correct answer is B. Many patients who are volume overloaded for various reasons, whether it is due to massive volume resuscitation or congestive heart failure, will develop some degree of scrotal edema. Often times, it can be very impressive and can also cause the patient pain. This patient definitely has signs of heart failure and has likely been heavily volume loaded, because of his sepsis. The only real treatment is to optimize the patient’s volume status and let the body reabsorb the fluid with diuresis as tolerated.

There are no signs of infection mentioned in the clinical scenario, thus treatment with an antibiotic is not necessary (choice A). Occasionally these patients will get some cellulitic type symptoms in the scrotum, but the treatment of choice for that would be something other than ciprofloxacin.

No specific treatment would help (choice C) is incorrect as he should be treated with diureses to improve his volume status.

The hydroceles seen in this patient are not likely to be contributing to the markedly increased size of the testicles, and percutaneous aspiration (choice D) is not necessary. Placing a drainage tube in the scrotum (choice E) is completely unnecessary and not done.

A 27-year-old heavy vehicle driver comes to the office because he is “not feeling well and has been losing weight” during the past few months. He also reports that he is feeling increasingly tired. He drives long hours on his job, smokes heavily, and admits to “moderate” amounts of alcohol intake. He has never seen a doctor before and denies any past medical or surgical history. His temperature is 37.0 C (98.6 F), blood pressure is 110/80 mm Hg, pulse is 70/min, and respirations are 16/min. Abdominal examination shows a vague abdominal mass in the midline that is not pulsatile and non-tender. Rectal examination is unremarkable. Scrotal examination shows an enlarged right testicle without sensation. The factor in this patient’s history and examination that is most helpful for diagnosing the etiology of the abdominal mass is

  A. alcohol intake
  B. his job
  C. non-pulsatile nature of abdominal mass
  D. scrotal examination findings
  E. smoking
Explanation:

The correct answer is D. Loss of testicular sensation and enlarged testis is diagnostic of testicular carcinoma. In advanced stages, testicular carcinoma spreads by lymphatics to the paraaortic lymph nodes. These lymph nodes are palpable in the midline of a thin person without any prominent pulsations.

Alcohol (choice A) and smoking (choice E) have no diagnostic value in evaluating paraaortic lymphadenopathy.

Increased temperature in the scrotum for a long time as in heavy vehicle long distance drivers may predispose them to testicular carcinoma. Job history (choice B) by itself is not diagnostic, hence it is incorrect.

Palpable, non-pulsatile paraaortic lymph nodes (choice C) may be seen with other cancers, but are not diagnostic of the specific primary pathology.

A 29-year-old man comes to the clinic because he and his wife have “not been able to have a baby”. The patient states that he has been happily married for 4 years and he and his wife have been trying to have a child for the last 13 months. He has never fathered a child and his wife has never been pregnant. His wife has been evaluated by her physician and no abnormalities were identified. Your patient denies any history of cryptorchidism, sexually transmitted diseases, urinary tract infections, genital trauma, or erectile dysfunction. He has not received any chemotherapy nor does he have any known genetic disorders. Physical examination reveals a circumcised phallus without meatal discharge. Testicles are descended bilaterally, and are normal in size and contour. There is a grade 3 varicocele on the left side. No varicocele is identified on the right. On rectal examination the prostate is normal to palpation. Serum testosterone, LH, and FSH are normal. You send the patient for semen analysis. The results are as follows

At this time you should

  A. advise them that no intervention is indicated
  B. obtain a transrectal ultrasound
  C. perform a testicular biopsy
  D. prescribe a testosterone patch
  E. refer them for in vitro fertilization
  F. refer him for a varicocelectomy (ligation of varicocele)
Explanation:

The correct answer is F. A varicocele is defined as a dilated vein or set of veins in the pampiniform plexus in the spermatic cord, and is the most common identifiable cause of male factor infertility. It is present in 15% of the total male population, but is found in approximately 40% of men with male factor infertility. A varicocelectomy (the ligation of varicoceles) improves semen quality in approximately two-thirds of men and doubles the chance of conception. Varicoceles form secondary to incompetent or absent valves in the spermatic veins. This valvular deficiency, combined with the long vertical course of the internal spermatic vein on the left side, leads to the formation of most varicoceles on the left side. A unilateral right sided varicocele suggests venous thrombosis (from a tumor) in the inferior vena cava. The effect of a varicocele on fertility has to do with the prevention of efficient blood flow out of the scrotum. There is pooling of blood in the pampiniform plexus, leading to an increase in scrotal temperature and an adverse effect on spermatogenesis. Varicoceles tend to cause a “stress pattern” on semen analysis. This is characterized by a low sperm concentration, low sperm motility, low sperm count, and low sperm morphology. The technique for varicocelectomy is varied and may be performed via an inguinal, retroperitoneal, subinguinal, laparoscopic, or embolization approach. The subinguinal approach with aid of a microscope, (microscopic varicocelectomy), is the approach with the fewest complications.

No intervention (choice A) is incorrect. Any couple who has been unable to conceive for over 1 year warrant investigation. In general, 90% of normal couples conceive within 1 year of trying. Some investigators advocate a simple, basic, cost-effective evaluation of both male and female at the time of presentation, no matter how long they have been attempting to conceive.

A transrectal ultrasound (choice B) is a valuable diagnostic tool when there is an obstructive process causing infertility. The majority of semen volume comes from the prostate, seminal vesicles, and Cowper’s gland. The majority of ejaculated sperm comes from the distal epididymis. Therefore, the normal volume of ejaculate and the fact that the patient is not azoospermic (no sperm present) makes obstruction of the vas, seminal vesicles, and ejaculatory ducts, unlikely. The patient also has a normal semen pH which is caused by the fructose within the semen. Fructose is secreted by the seminal vesicles, therefore this patient’s normal semen pH makes seminal vesicle obstruction unlikely. In this scenario, the yield of transrectal ultrasound to look for seminal vesicles or ejaculatory duct dilatation, is low.

Testicular biopsy (choice C) is indicated in azoospermic patients with normal FSH levels or in patients with abnormal hormone parameters. This patient has neither.

The patient has normal hormone parameters, and there is no evidence that increasing his testosterone (choice D) will improve his semen parameters.

Referring this couple for in vitro fertilization (choice E) is not the correct management. The male in the relationship has a potentially reversible cause of his infertility (varicocele) and having them go through the emotional and financial hardship of assisted reproduction is not recommended prior to attempting correction of other causes of infertility.

A 54-year-old African American man comes to the office complaining of swelling in his left scrotum. He states that the swelling has slowly gotten worse over the past 6 months and he can no longer feel his left testicle. As per the patient, the swelling itself does not cause pain. However, the swollen scrotal skin is rubbing against his thigh causing an irritation. The patient’s urologic history is significant for 2 episodes of epididymitis in the past 5 years. He denies any trauma to the scrotum, dysuria, hematuria, infertility, or prior similar episodes. There are no constitutional symptoms elicited with further questioning. The patient is afebrile and on examination the left hemi-scrotum is obviously enlarged and the scrotal skin is tense. There is no erythema of the scrotum. The left testicle is non-palpable. You are able to transilluminate light through the left scrotal mass. The mass is not reducible through the inguinal ring and it does not change in size or consistency with Valsalva or when the patient lies down. The right testicle is descended and normal to palpation. There is no urethral discharge, inguinal adenopathy, or abnormalities on rectal examination. Urinalysis and laboratory values are normal. The most likely underlying cause of this patient’s scrotal swelling is

  A. dilatation of the pampiniform plexus
  B. fluid collection within the tunica vaginalis
  C. renal-cell carcinoma with invasion into the left renal vein
  D. testicular neoplasm
  E. torsion of the spermatic cord
Explanation:

The correct answer is B. This patient has a hydrocele. A hydrocele is a collection of fluid within the tunica (or processus) vaginalis. The diagnosis is made by finding a rounded cystic intrascrotal mass that is not tender unless underlying inflammatory disease is present. The mass transilluminates, helping to differentiate it from a testicular neoplasm. If the fluid is allowed to continue to build up, then it may eventually become large enough to prevent appropriate evaluation of the testicle. If a hydrocele develops in a young man, without apparent cause, then careful evaluation of the testicle and epididymis should be done in order to rule out cancer or infection. If the exact diagnosis is in question, then a scrotal ultrasound should be performed.

Varicocele is caused by dilatation of the pampiniform plexus (choice A). It is more common on the left side. On exam these patients have a mass of dilated, tortuous veins lying posterior to and above the testis. It may extend up to the external inguinal ring and is often tender. The degree of dilatation can be increased by the Valsalva maneuver. In the recumbent position, venous distention disappears. The sudden development of a varicocele in an older man is sometimes a late sign of a renal tumor that has invaded the left renal vein, thereby obstructing left spermatic vein drainage (choice C).

Testicular neoplasm (choice D) as previously stated does not transilluminate. Also, the history associated with a testicular tumor usually involves a dull, heavy feeling within the testicle. These masses have the potential to grow rapidly. Some testicular tumors are associated with a small hydrocele. With a testicular tumor, the testicle is hard, enlarged, and irregular in shape.

Testicular torsion (choice E) occurs when the spermatic cord twists, causing strangulation of the blood supply to the testis. Examination reveals a swollen, tender testicle that is retracted upward. It is more common in young boys who develop acute testicular pain.

 

A 68-year-old man is admitted to the hospital for intravenous antibiotic therapy for cellulitis of the elbow. After 3 days of therapy, he is feeling better, but now complains of difficulty walking. He has a known history of prostate cancer that was treated with radical retropubic prostatectomy 6 years ago. His recovery from surgery was uneventfu,l and initially his prostate specific antigen (PSA) remained undetectable. Because he had been feeling well, he stopped his follow up with his urologist. He has received no treatment for his cancer since his surgery. He has not had a complete urologic examination in over 2 years. The patient states that approximately 2 days prior to admission, he noticed that he had some mild difficulty lifting his left leg off the ground. His symptoms did not improve, and today he developed weakness of the right leg as well. He denies any trauma, falls, loss of bowel or bladder control or prior similar episodes. However, he does state that his lower back has been “bothering” him for the last 2 months. Vital signs are within normal limits. Physical examination shows good anal sphincter tone, and an empty prostatic fossa. Neurologic examination demonstrates decreased motor sensation of the lower extremities below the L2 level. Sensation is diminished below the L2 level as well. Plain films of the spine do not display any osteoblastic lesions. Serum PSA level is 726 ng/mL. The hormonal treatment that may prevent further neurologic deterioration in the shortest period of time is

  A. bilateral adrenalectomy
  B. bilateral orchiectomy
  C. intravenous diethylstilbestrol (DES)
  D. ketoconazole
  E. luteinizing hormone releasing hormone (LHRH) agonist
Explanation:

The correct answer is B. This patient is suffering from spinal cord compression secondary to metastatic prostate cancer. This is a medical emergency, and if left untreated may lead to significant morbidity, including paraplegia and autonomic dysfunction. Pain usually precedes the diagnosis of spinal cord compression by up to 4 months. However, symptoms can progress rapidly to neurologic dysfunction in a matter of hours to days. The combination of the patient’s history, clinical exam, and dramatically elevated PSA level confirm his diagnosis. The diagnosis of spinal cord compression is usually made with a combination of physical exam, plain films, and bone scans. Once the diagnosis is made, immediate androgen deprivation is required (assuming the patient has never been treated with hormones). The goal with androgen deprivation is to “starve” the prostate cancer of the hormone testosterone, which it uses to flourish. Once the diagnosis of spinal cord compression is made and androgen deprivation is begun, the patient should also be placed on steroid therapy. Steroids help to decrease vasogenic edema as well as provide analgesic benefit. Other treatment modalities may be required in conjunction with androgen deprivation and steroids. These include radiation and possible surgery for those patients with spinal instability. Bilateral orchiectomy is permanent. It removes all testicular production of testosterone, which is more than 90% of total testosterone produced by men. Castrate levels of serum testosterone are achieved in approximately 3 hours after surgery.

Bilateral adrenalectomy (choice A) will only remove the adrenal production of testosterone (approximately 10% of total testosterone production). Not only is this not adequate androgen deprivation for this patient, but it is no longer necessary now that pharmacological agents are available.

Diethylstilbestrol (DES) (choice C) is a synthetic, non-steroidal form of estrogen. In 24 hours, it only causes a 50% reduction in total testosterone. DES has many side effects including pulmonary embolism, myocardial infarction, and deep venous thrombosis.

Ketoconazole (choice D) is a direct inhibitor of testosterone production. It works by blocking cytochrome P-450, thereby directly halting the production of both adrenal and gonadal testosterone. Castrate levels of testosterone are produced in 8 hours. This is a good alternative in patients who are unfit for surgery, refuse surgery, or will not be able to get to the operating room in an expeditious manner.

Luteinizing hormone-releasing hormone (choice E) medications work by agonizing LHRH receptors in the pituitary. They produce an initial increase in release of LH and FSH causing an increase in testosterone production (“flare phenomenon”). Therefore, they are contraindicated in this setting, because the increase in testosterone will make the patient’s condition worse. With time, these medications suppress LH and FSH production by inhibiting the hypothalamic-pituitary axis. Ultimately, this leads to a decrease in testosterone production. With these medications, testosterone reaches castrate levels in 30 days.

 

A 54-year-old African American man comes to the office complaining of swelling in his left scrotum. He states that the swelling has slowly gotten worse over the past 6 months and he can no longer feel his left testicle. As per the patient, the swelling itself does not cause pain. However, the swollen scrotal skin is rubbing against his thigh causing an irritation. The patient’s urologic history is significant for 2 episodes of epididymitis in the past 5 years. He denies any trauma to the scrotum, dysuria, hematuria, infertility, or prior similar episodes. There are no constitutional symptoms elicited with further questioning. The patient is afebrile and on examination the left hemi-scrotum is obviously enlarged and the scrotal skin is tense. There is no erythema of the scrotum. The left testicle is non-palpable. You are able to transilluminate light through the left scrotal mass. The mass is not reducible through the inguinal ring and it does not change in size or consistency with Valsalva or when the patient lies down. The right testicle is descended and normal to palpation. There is no urethral discharge, inguinal adenopathy, or abnormalities on rectal examination. Urinalysis and laboratory values are normal. The most likely underlying cause of this patient’s scrotal swelling is

  A. dilatation of the pampiniform plexus
  B. fluid collection within the tunica vaginalis
  C. renal-cell carcinoma with invasion into the left renal vein
  D. testicular neoplasm
  E. torsion of the spermatic cord
Explanation:

The correct answer is B. This patient has a hydrocele. A hydrocele is a collection of fluid within the tunica (or processus) vaginalis. The diagnosis is made by finding a rounded cystic intrascrotal mass that is not tender unless underlying inflammatory disease is present. The mass transilluminates, helping to differentiate it from a testicular neoplasm. If the fluid is allowed to continue to build up, then it may eventually become large enough to prevent appropriate evaluation of the testicle. If a hydrocele develops in a young man, without apparent cause, then careful evaluation of the testicle and epididymis should be done in order to rule out cancer or infection. If the exact diagnosis is in question, then a scrotal ultrasound should be performed.

Varicocele is caused by dilatation of the pampiniform plexus (choice A). It is more common on the left side. On exam these patients have a mass of dilated, tortuous veins lying posterior to and above the testis. It may extend up to the external inguinal ring and is often tender. The degree of dilatation can be increased by the Valsalva maneuver. In the recumbent position, venous distention disappears. The sudden development of a varicocele in an older man is sometimes a late sign of a renal tumor that has invaded the left renal vein, thereby obstructing left spermatic vein drainage (choice C).

Testicular neoplasm (choice D) as previously stated does not transilluminate. Also, the history associated with a testicular tumor usually involves a dull, heavy feeling within the testicle. These masses have the potential to grow rapidly. Some testicular tumors are associated with a small hydrocele. With a testicular tumor, the testicle is hard, enlarged, and irregular in shape.

Testicular torsion (choice E) occurs when the spermatic cord twists, causing strangulation of the blood supply to the testis. Examination reveals a swollen, tender testicle that is retracted upward. It is more common in young boys who develop acute testicular pain.

A 24-year-old man comes to the emergency department with a 3–day history of urethral discharge and burning when he urinates. He initially noticed a milky discharge from the penis, that is now more yellowish in color. He has no known drug allergies. He is sexually active with multiple different partners per month. His temperature is 37 C (98.6 F). A yellowish discharge can be expressed from the urethral meatus. A digital rectal examination demonstrates a normal feeling prostate gland without tenderness. Laboratory studies are remarkable for a leukocyte count of 8,000mm3, creatinine of 0.9 mg/dL, and blood urea nitrogen of 16 mg/dL. A urethral swab shows multiple white blood cells with Gram-negative intracellular diplococci. The most appropriate treatment for this patient is

  A. azithromycin 1 gm orally once
  B. ceftriaxone 125 mg intramuscularly once
  C. ceftriaxone 125 mg intramuscularly once plus azithromycin 1 gm orally once
  D. ciprofloxacin 500 mg by mouth twice daily for 14 days
  E. trimethoprim-sulfamethoxazole 1 double strength tablet twice daily for 14 days
Explanation:

The correct answer is C. This patient is presenting with a classic case of gonococcal urethritis. There is a yellowish discharge, which can be expressed from the penis and Gram stain shows intracellular Gram-negative diplococci. Thus the main question left to answer is how should this be treated. One important thing to know is that there are many choices in therapy. However, all therapy should address not only the gonococcal infection, but also the possibility of a chlamydial infection. Concomitant C. trachomatis occurs in up to 50% of patients. The ceftriaxone alone (choice B) would be adequate for the gonorrhea, but the azithromycin should be added for chlamydial treatment. Similarly azithromycin alone (choice A) is not enough. Another choice besides azithromycin would be doxycycline. However, it requires a 10-day course of therapy. One other important issue in these cases, is that the partner of the person should be treated as well, or else the two will continue to infect each other.

Ciprofloxacin (choice D) can be used in the treatment of gonococcal urethritis, but only a one time dose is required, and it too does not address the possibility of a chlamydial infection.

Trimethoprim-sulfamethoxazole (choice E) is not effective in this disease.

 

 

A 58-year-old man comes to the office because of difficulty with erections for the past few years. He says that he has a great relationship with his wife and is still very sexually aroused by her. He is occasionally able to initiate an erection, but he is unable to sustain it. The remainder of his medical, sexual, and psychological history is unremarkable. He takes isosorbide mononitrate for chest pain. His blood pressure is 130/90 mm Hg. Physical examination is unremarkable. Prolactin and testosterone levels are within normal limits. He asks for the “little blue pill” that is so often advertised on television commercials. At this time you should

  A. advise him to discontinue the isosorbide mononitrate
  B. explain that he cannot take sildenafil because of his current medication
  C. prescribe sildenafil citrate tablets for him to take an hour before sexual activity
  D. recommend implantation of an inflatable prosthesis
  E. tell him that his erectile dysfunction is psychogenic
Explanation:

The correct answer is B. Erectile dysfunction (ED) is a very common problem. The evaluation typically includes a detailed history and physical examination, serum testosterone and prolactin levels, and a medication review. This patient’s evaluation is unremarkable. He is specifically asking for a prescription for sildenafil, but it is contraindicated in patients taking any medications that contain nitrates because the combination can lead to life-threatening hypotension, a heart attack, or stroke. This should be explained to him and other options should be considered. Sex therapy and/or a vacuum device (that draws blood into the penis) should be offered.

Since this patient is taking isosorbide mononitrate for angina, which can be a serious problem, he should not be advised to discontinue it (choice A). Also, this agent is not typically associated with ED. Some cardiovascular drugs that have been associated with ED are thiazides, spironolactone, calcium channel blocks, methyldopa, beta blockers, clonidine, and digoxin.

Since this patient takes a nitrate medication, it is incorrect to prescribe sildenafil citrate tablets for him to take an hour before sexual activity (choice C). Sildenafil is contraindicated in patients taking nitrates.

Since there are other less invasive options for this patient to try, it is inappropriate to recommend implantation of an inflatable prosthesis (choice D) at this time. Sex therapy and a vacuum device should be offered.

It is incorrect to tell him that his erectile dysfunction is psychogenic (choice E) because a full evaluation was not performed. Nocturnal penile tumescence testing, vascular testing, neurologic testing, and psychological testing can help determine the exact cause. All of these tests are not usually necessary because they may be expensive and invasive.

 

A 32-year-old man comes to the clinic with complaints of “swelling in his left testicle”. He states that for the past several months he has noticed a lump in his left hemiscrotum. He denies any pain or tenderness. He denies erythema and there has been no fever, dysuria, or urgency. He is married and claims to be monogamous. His temperature is 37 C (98.6 F), blood pressure is 120/80 mm Hg, pulse is 60/min, and respirations are 16/min. His abdominal examination is benign. The scrotum appears normal. However, on palpation there is fullness in the left hemiscrotum. A urinalysis is normal. An ultrasound shows normal appearing testes bilaterally and a moderately sized left hydrocele. The most appropriate next step in management is to

  A. admit the patient for drainage of the fluid collection
  B. explain that no specific treatment is necessary at this time but he should follow up in 6 months
  C. order a CT scan of the abdomen and pelvis to rule out other abnormalities
  D. prescribe ciprofloxacin 500 mg twice daily for 2 weeks and then reassess
  E. refer the patient to urology for an orchiectomy
Explanation:

The correct answer is B. The diagnosis in this case is easily made with an ultrasound. A hydrocele is simply a fluid collection between the layers of the tunica vaginalis. They are typically idiopathic and require no treatment. Hydroceles can fluctuate in size, which typically means that they are in some communication with the peritoneal cavity, and thus represent an indirect inguinal hernia. If it does persist or does represent an indirect hernia, referral to urology for surgical evaluation is warranted. There is certainly no need at this time for acute treatment (choice A).

CT scan of the abdomen and pelvis (choice C) would not be indicated in this case as the patient’s symptoms are referable directly to a palpable abnormality in the testicle. There are no signs or symptoms of intraabdominal pathology at this time.

Ciprofloxacin (choice D) is commonly used in genitourinary tract infections. This patient has no signs of infection at this time. Sometimes in the setting of an epididymitis, a hydrocele with particulate matter within it, will be present and be seen on ultrasound examination. This is referred to as a pyocele and may need to be drained surgically, much like an abscess would, in order to adequately treat this patient’s infection.

An orchiectomy (choice E) would be performed for testicular malignancies or possibly in a torsed testicle that is necrotic.

 

A 51-year old married man comes to the office complaining of blood in his semen. He states that approximately 2 weeks prior to presentation he noticed bloody ejaculate. There were 2 episodes within 3 days of each other. Since the last episode he has had normal ejaculations on multiple occasions. There is no associated pain, penile discharge, erectile dysfunction, abdominal pain, or history of trauma. His medical history is significant for diet-controlled diabetes and eczema. There is no family history of prostate cancer. Physical examination reveals no abnormalities of the penis or scrotum. On digital rectal examination, his prostate is smooth, non-tender, firm, normal in size, and without nodule. Serum prostate specific antigen (PSA) is 1.4 ng/mL. Urinalysis and urine cytology are both negative. The next best step in management is

  A. observation and reassurance
  B. obtain a semen analysis
  C. order a transrectal ultrasound
  D. perform cystoscopy or cystourethroscopy
  E. prescribe ciprofloxacin for 7 days
  F. repeat PSA
  G. schedule a prostate biopsy
Explanation:

The correct answer is A. Hematospermia, the presence of blood in the seminal fluid, is usually the result of nonspecific inflammation of the urethra, prostate, or seminal vesicles. It is almost always self-limiting and resolves within several weeks. Occasionally, hematospermia may be associated with infection, particularly tuberculosis, cytomegalovirus, and schistosomiasis, but rarely is it associated with malignancy. All patients with this complaint should undergo careful physical exam to exclude hypertension which may cause hematospermia, a rectal exam, and a PSA test to exclude prostatic carcinoma, and a urine cytology to rule out the possibility of transitional cell carcinoma of the prostate. This patient has had a complete work-up for initial presentation of hematospermia. There is no need to do anything but reassure the patient that his condition is almost always self-limiting.

Semen analysis (choice B) evaluates semen for volume and sperm concentration, quantity, motility, and morphology. Its utility is in the evaluation of infertility.

Transrectal ultrasonography (choice C) can evaluate the prostate, seminal vesicles, or ejaculatory ducts. It may be utilized for persistent hematospermia. It does not play a role in evaluation of a single episode of hematospermia.

Cystoscopy, or cystourethroscopy (choice D), allows for visualization of the pendulous, bulbar and prostatic urethra and bladder, and is not indicated unless hematospermia persists and there is concern for the underlying pathology.

Antibiotics (choice E) can be utilized in the treatment of prostatitis. There is no evidence that reveals utility of such agents in the current clinical scenario. When treating prostatitis, antibiotics should be used for at least 21 days. Patients with prostatitis normally have a soft, boggy, and tender prostate on rectal exam and have associated urinary complaints (dysuria and frequency).

Prostate specific antigen (PSA) (choice F) is a useful screening tool for prostate cancer. In general, a normal PSA is any value less than 4. Some urologists also believe in age-adjusted PSA with a normal value being slightly lower for younger patients. A repeat exam is not necessary in light of this patient’s normal level.

Prostate biopsy (choice G) is an invasive procedure that is performed in patients who have a clinical suspicion of prostate cancer (i.e., abnormal digital rectal exam or elevated PSA). This patient has neither, and subjecting him to an unnecessary invasive test is not indicated.

 

 

An 18-year-old man comes to the clinic complaining of heaviness in his left testicle. He noticed this for the first time 3 weeks ago after “pulling his groin” in a high school football game. The groin pull has improved but the discomfort in the testicle has not. He also states that he has noticed the left testicle is larger than the right testicle. His pain is non-radiating, dull in character, and not associated with any dysuria or discharge. He admits to an episode of unprotected intercourse with a new partner approximately 1 month ago. There is no weight loss, fever, cough, or headaches. Physical examination is significant for a left testicle that is non-tender, hard, increased in size as compared to the right, irregular in contour, and without transillumination. There is no inguinal adenopathy. The right testicle is normal in size and shape. No discharge is expressed per urethra. Urinalysis and urine culture are negative. Beta-human chorionic gonadotropin (bHCG) level and alpha-fetoprotein (AFP) levels are normal. The next most appropriate management for this condition is

  A. to administer chemotherapy
  B. external beam radiation therapy to scrotum
  C. external beam radiation therapy to retroperitoneum
  D. left radical orchiectomy via a scrotal incision
  E. left radical orchiectomy via an inguinal incision
  F. open testicular biopsy
  G. prescribe antibiotics
  H. recommend scrotal support, antiinflammatory drugs, and reevaluation in 2 weeks
  I. schedule incision and drainage of scrotal abscess
Explanation:

The correct answer is E. This patient has testicular cancer until proven otherwise. The most common symptom of testicular cancer is painless enlargement of the testis. Patients frequently complain of a sensation of testicular heaviness. 10% of patients will present with acute testicular pain as a result of intratesticular hemorrhage or infarction and 10% of patients will present with symptoms related to metastatic disease (back pain from retroperitoneal metastases involving nerve roots, cough or dyspnea from pulmonary metastases, etc). And 10% of patients are asymptomatic at presentation and the mass may be picked up incidentally following trauma or by the patient’s sexual partner. An incorrect diagnosis is made at the initial examination in up to 25% of patients with a testicular tumor and may result in delay in treatment or surgery via a scrotal approach. It is important not to violate the scrotal skin in the presence of testicular cancer. The lymphatic drainage of the testis is the retroperitoneum, while the scrotal skin drains via the inguinal nodes. Violating the scrotal skin has the potential to extend the spread of metastases outside the normal anatomic path. This patient’s symptoms and physical findings are classic for testicular carcinoma. While bHCG and AFP may be elevated in a variety of testicular cancers, they do not have to be. The normal levels in this patient should not discourage one from making the proper diagnosis. Epididymitis or epididymoorchitis are the most common misdiagnoses in patients with testis cancer. These patients typically have an enlarged tender epididymis that is separable from testicle. In advanced stages the inflammation may spread to the testis and enlarge the testicle as well. There is usually an acute onset of symptoms associated with fever, urethral discharge, and irritative voiding symptoms. Hydrocele is also a common misdiagnosis. Transillumination of the scrotum may reveal a translucent, fluid-filled hydrocele versus a solid tumor. Since approximately 5-10% of testicular tumors may be associated with hydroceles, if there is any question, then a scrotal ultrasound is mandatory.

Further therapy of testicular cancer depends on the type of tumor and clinical stage. Chemotherapy (choice A) and retroperitoneal external beam radiation therapy (choice C) both play a role in the adjuvant treatment of testicular cancer. However, neither is appropriate management prior to radical orchiectomy.

Radiation therapy to the scrotum plays no role in the treatment of testicular cancer (choice B).

Initial treatment for all testicular cancer or presumed cancer is inguinal exploration and high ligation of the spermatic cord with removal of the testicle and spermatic cord (radical orchiectomy). As previously stated, scrotal approaches (choice D) and open testicular biopsy (choice F) should be avoided so as not to disrupt the lymphatic drainage system.

Antibiotics (choice G), scrotal support, and antiinflammatories (choice H) are all appropriate therapy for patients with epididymitis or epididymoorchitis. As previously explained, this patient does not have these diagnoses and providing this form of treatment will only delay appropriate care.

Drainage of an abscess (choice I) is appropriate for any patient who has this diagnosis. Patients with scrotal abscess have swelling, tenderness, erythema, fluctuance, and possibly fever. This patient does not have any of these symptoms, making this diagnosis extremely unlikely.

 

A 65-year-old man comes to the office because he constantly feels like he needs to urinate, even after he just went. He states that over the past few months, he has been waking up a few times a night to urinate, and he needs to “push very hard to get that urine out.” The urinary stream is typically weak, and he turns red when he says that he often “dribbles” when he is done. You have been treating him for typical “colds”, “backaches”, and gout over the years, and lately you have been monitoring his blood pressure, which has ranged from 140/90 mm Hg to 150/90 mm Hg in the past 8 months. He has been the “ideal patient.” He started a moderate exercise program, quit smoking, eliminated all alcohol, and cut down on salt and fat, but his blood pressure has remained elevated. All studies, including a urinalysis, complete blood count, electrolytes, BUN and creatinine, cholesterol, glucose, plasma uric acid, chest x-ray, and electrocardiogram were normal at the time of the initial hypertension work-up. His temperature is 37 C (98.7 F), blood pressure is 145/85 mm Hg, and pulse is 65/min. Digital rectal examination shows an enlarged, prostate gland. Funduscopic examination and urinalysis are normal. His prostate-specific antigen is 3 ng/mL. You discuss treatment options for his conditions, and he decides that he wants to take the “least amount of pills possible.” Based on his statement, the most appropriate pharmacotherapy at this time is

  A. enalapril
  B. finasteride
  C. hydrochlorothiazide
  D. nifedipine
  E. terazosin
Explanation:

The correct answer is E. This patient has benign prostatic hyperplasia (BPH) and hypertension, which can both be treated with terazosin. Terazosin is a long acting selective alpha-1 adrenergic blocker that is useful in treating essential hypertension and symptomatic benign prostatic hyperplasia. The symptoms of BPH are typically due to bladder outlet obstruction and urinary stasis. Terazosin most likely provides symptomatic relief of BPH by antagonizing the contraction of the bladder sphincter and relaxing the smooth muscle of the bladder neck. It is an effective treatment for hypertension because of its vasodilatory properties.

Enalapril (choice A) is an angiotensin-converting enzyme inhibitor that is useful in treating hypertension, but not BPH.

Finasteride (choice B) is a 5-alpha reductase inhibitor that blocks the conversion of testosterone to dihydrotestosterone, and is used to treat BPH, not hypertension.

Hydrochlorothiazide (choice C) is a diuretic that is used to treat hypertension, but has no effect on BPH. It may actually be uncomfortable for him to have to urinate even more than he is already. Also, thiazides cause hyperuricemia, which can lead to gout. This patient has a history of gout, which is a reason to not give him thiazides.

Nifedipine (choice D) is a calcium-channel blocker that is used to treat hypertension, but is not effective for BPH.

 

A 76-year-old man with diabetes and hypertension is admitted to the hospital for intravenous antibiotic therapy to treat pneumonia. He had been improving during the first few days he was in the hospital. However, 5 days later, he is now having problems with urinary retention. His Foley catheter was removed 24 hours ago and the patient is unable to void. This morning the nurse reinserted a catheter, which drained 900 cc of cloudy urine. Tonight, the patient began complaining that the catheter bothers him and he keeps pointing to his penis. You ask the nurse appropriate questions and learn that he is and has been afebrile, and is currently completing a course of cephalosporins for his pneumonia. The nurse who placed the catheter is no longer in the hospital, but by report, there was no difficulty with Foley catheter insertion. Over the last 12 hours, the patient has drained 750 cc of urine. Upon entering the patient’s room, you see an elderly man who is obviously uncomfortable. He states that the catheter really hurts and he has never felt anything like this before. He denies any abdominal pain, stating that all the pain is at the point where the catheter enters the penis. The patient tells you that he has never been circumcised. On physical examination, his abdomen is soft and non-distended without any suprapubic discomfort. Examination of his penis shows that the glans is exposed, edematous, red and tender to touch. At the level of the coronal sulcus is a piece of edematous tissue that looks as though a ring has been placed over his penile shaft. The proximal aspect of the penis is also swollen, but not to the degree of the tissue at the coronal sulcus. His testicles are descended bilaterally, and there is mild tenderness over the right epididymis. Rectal examination reveals an enlarged prostate with a hard, raised nodule over the right base. The most appropriate next step in this patient’s management is to

  A. adjust his antibiotics for better urinary coverage
  B. obtain a scrotal ultrasound
  C. order PSA
  D. reduce the foreskin
  E. remove the Foley catheter
  F. schedule a prostate biopsy
  G. send for a urinalysis and culture
Explanation:

The correct answer is D. This patient is suffering from a paraphimosis. This occurs in uncircumcised males who have their foreskin retracted beyond the coronal sulcus, so that it is not subsequently reduced. This has the potential to become an emergency, as the retracted foreskin will act as a tourniquet around the penis, leading to pain, edema, and possibly, vascular compromise. With time the penis will appear as though there is a ring around the distal aspect of it. Important components to the diagnosis of this condition include history and physical. A high level of suspicion is necessary and asking the patient about a history of circumcision is very important. Treatment of this urologic emergency involves attempted manual reduction. This is done with manual compression of all edema out of the glans followed by reduction of the foreskin. If this cannot be accomplished, then the patient will need surgical correction with either a dorsal slit or circumcision.

This scenario is a perfect example of why a physical exam is so important. While Foley catheters may be uncomfortable, they should not be excruciatingly painful once in appropriate position. Changing antibiotics (choice A) does not address the emergent issue. Urinary tract infection may cause urinary retention as this patient had. If one is concerned for UTI, then a urinalysis and urine culture (choice G) would be appropriate after reduction of the foreskin.

The mild epididymal pain, the patient has, may be a bout of epididymitis, which can happen in patients with an in-dwelling catheter. Obtaining a scrotal ultrasound (choice B) will help in this diagnosis but plays no role in evaluation and treatment of a paraphimosis.

A PSA (choice C) and prostate biopsy (choice F) are both utilized in the work-up of prostate cancer. A hard nodule on prostate exam, does raise the suspicion for prostate cancer. However, these tests can be performed at a later date.

Removing this patient’s catheter (choice E) is the wrong management for paraphimosis. The foreskin can be reduced with the catheter in place. Physical exam is so important in this case, or else the diagnosis will be missed. Besides, it is unlikely that this patient will be able to void on his own so soon after his bladder was allowed to distend with 900 cc of urine.

 

 

A 43-year-old Caucasian man comes to the emergency department because of a 6-hour persistent erection. He complains of some discomfort within the penis, but denies any trauma associated with intercourse. He does have some dysuria. He has never experienced anything similar to this and his facial expression displays his concern. He has no significant medical history, and is not taking any medication, prescribed or illicit. On physical examination, the corpora cavernosum is rigid and tender, while the glans penis and corpus spongiosum are soft. There is no curvature associated with the erection, and there are no palpable abnormalities along the length of the penile shaft. The testicles are descended and normal to palpation. Rectal examination reveals good sphincter tone and a normal sized, smooth prostate. Urinalysis and blood counts are all within normal limits. The most likely diagnosis is

  A. epididymitis
  B. hypospadias
  C. paraphimosis
  D. penile chordee
  E. penile fracture
  F. Peyronie’s disease
  G. priapism
Explanation:

The correct answer is G. This patient is suffering from priapism. Priapism is defined as an abnormally prolonged, and usually painful, erection that does not result from sexual desire. It generally involves only the corpora cavernosum and not the corpora spongiosum. The disorder is idiopathic in 60% of the cases, while the remaining 40% of cases are associated with diseases (i.e., leukemia, sickle-cell disease, pelvic tumors, and pelvic infections). The patient usually presents with a history of several hours of painful erection. As in this patient, the glans penis and corpus spongiosum are soft and uninvolved in the process. The corpora cavernosa are tense with congested blood and tender to palpation. Priapism can be classified as low-flow or high-flow. High-flow is associated with trauma, is non-ischemic, and non-painful. The penis is not fully rigid and the cause is due to unregulated arterial inflow. Low-flow priapism is associated with ischemia, veno-occlusion and stasis of blood, resulting in a painful, rigid erection. There is a build-up of highly viscous, poorly oxygenated blood within the corpora cavernosa. If this persists then interstitial edema and fibrosis of the corpora cavernosa will develop and ultimately lead to impotence.

Epididymitis (choice A) is due to infection and inflammation of the epididymis. The patient complains of heaviness and discomfort in the affected hemi-scrotum.

Hypospadias (choice B) occurs when the urethral meatus opens on the ventral aspect of the penis and not the tip of the glans. It is due to abnormal development in utero.

Paraphimosis (choice C) is a urological emergency. It occurs when the foreskin is retracted and not replaced back into its normal position. The retracted foreskin acts as a tourniquet on the glans, and if it is not reduced, may lead to ischemia of the glans penis.

Penile chordee (choice D) is congenital fibrosis of the tunica albuginea. It leads to curvature of the penis. It is frequently seen in association with hypospadias, and may occur on the ventral (more common) or dorsal side of the penis.

Penile fracture (choice E) occurs with traumatic intercourse. A tear in the tunica albuginea, while the penis is erect leads to rapid detumescence with penile edema, pain, and hematoma.

Peyronie’s disease (choice F) is defined as an abnormal thickening (or plaque) of the tunica albuginea. It differs from chordee in that it is an acquired condition. It is usually associated with traumatic intercourse. It leads to painful erection, curvature of the penis, and poor erection distal to the involved area. Plaques are palpable, dense, varying in size, and may be located on the dorsal or ventral aspect of the penile shaft.

 

 

A 64-year-old man with hypertension, diabetes, and hyperlipidemia comes to your office complaining of difficulty achieving erection. His medications include nifedipine, simvastatin, losartan, metformin, and glyburide. He has been married for 30 years and he tells you that his wife is becoming frustrated with his “lack of interest in her.” He denies ability to achieve erection with self stimulation and no longer wakes up in the morning with an erection as he did when he was younger. He tells you that a friend informed him that there is a medication that can help with men who have this problem. Physical examination shows a moderately obese man with normal size testes. Digital rectal examination reveals a slightly enlarged, non-tender prostate without palpable nodules. The remainder of his examination is normal. Laboratory studies show:

The most likely underlying cause of his impotence is

  A. hormonal changes
  B. medications, especially nifedipine
  C. neuronal injury
  D. psychological factors
  E. vascular disease
Explanation:

The correct answer is E. The most common cause of erectile dysfunction in patients older than 50 is vascular disease. This patient has diabetes, hypertension, and high cholesterol so it is very likely that that he has a physical disruption of blood flow to his penis.

Hormonal causes of erectile dysfunction (choice A) include elevated prolactin levels, testicular failure, pituitary dysfunction, or abnormal thyroid. These disorders may be less likely in this patient since they typically cause a decreased libido, which this patient does not have. In a patient with normal testicular size, low testosterone levels would be less likely. If testosterone levels are low, it would be worthwhile to check LH and FSH levels to evaluate pituitary function.

Medications (choice B) are responsible for a large percentage of erectile dysfunction. Beta blockers and thiazide diuretics are often the culprits. Although all drugs should be suspected, calcium channel blockers and ACE inhibitors cause these problems less frequently. In studies, 3 % (or less) of patients taking nifedipine reported erectile dysfunction, which was the same number of men taking a placebo that complained of this problem.

Nerve injury (choice C) can result in erectile dysfunction in patients with pelvic surgery, pelvic trauma, spinal cord injury, MS, etc. This patient has no history leading us to suspect nerve damage as the etiology of his erectile dysfunction.

Psychological factors (choice D) are responsible for a small percentage of erectile dysfunction. If you suspect psychological factors are responsible for a patient’s erectile dysfunction, referral to a specialist is warranted. Also, since he is not able to achieve erection with self stimulation and no longer has morning erections, psychological causes of dysfunction are less likely.

 

 

A 19-year-old man comes to the clinic for a periodic physical examination. He has no complaints and no significant past medical history. He is on no medications and reports no allergies to medicines. Similarly, the family, social, and health risk history is unremarkable as well. You start a complete physical checkup and are surprised to find a third lump in his testicular region. On questioning, he tells you that he has noticed it before, but was too embarrassed to bring it up. A testicular ultrasound is performed and reveals the lump to be consistent with testicular cancer. To clarify the picture further, you order an alpha-fetoprotein (AFP) and the beta subunit of the human chorionic gonadotropin (hCG). The AFP level is normal, but the hCG level is elevated. A CT scan of the chest, abdomen, and pelvis show no retroperitoneal node involvement and no metastases to distant areas. Based on the findings above, you tell him that the tumor is most likely

  A. nonseminoma type, Stage 1, and a retroperitoneal lymph node dissection is indicated
  B. nonseminoma type, Stage 2, and chemotherapy is indicated
  C. seminoma type, Stage 1, and an orchiectomy with adjuvant radiation therapy is indicated
  D. seminoma type, Stage 2, and either radiotherapy or chemotherapy is indicated
Explanation:

The correct answer is C. Testicular cancer is the most common cancer in men 20-40 years of age. Lifetime incidence is .4% for white male and .08% in black males. Cryptoorchidism is the biggest risk factor for testicular cancer and it increases the risk about fourfold. Nearly 95% of all malignant testicular cancers are of the germ cell type that are divided into the seminoma and nonseminoma. Various tumor markers can help distinguish between the two types of germ cell tumors. Elevated AFP is produced by embryonal and yolk sac elements which are found only in nonseminomas. HCG however, is produced by both seminomas and nonseminomas. A lactate dehydrogenase (LDH) can also be used, but is less specific. Staging is done to assess the extent of disease and to guide therapy. Stage 1 cancer is confined to the testis, epididymis, or spermatic cord. Stage 2 is limited to the retroperitoneal nodes, subclassified by the size of the nodes into < 2 cm, between 2-5 cm, and > 5 cm in diameter. Stage 3 involves metastases to the supradiaphragmatic nodes or visceral sites. Based on the above information, this patient has a seminoma type, Stage 1 process. The standard of therapy is postorchiectomy adjuvant radiotherapy. Since there is no retroperitoneal node involvement, his case is not stage 2 (choice D). If it was stage 2, radiotherapy or chemotherapy can usually cure the cancer.

Since the patient’s cancer does not secrete AFP, his cancer is not of the nonseminoma type. Staging was described previously. In general, retroperitoneal lymph node dissection is the standard of care for patients with Stage 1 nonseminoma cancer (choice A). Stage 2 (choice B) and 3 disease requires early retroperitoneal lymph node dissection and either close conservative surveillance or immediate chemotherapy afterwards.

 

 

A 34-year-old man comes to the office because of “erectile problems.” He says that he and his wife have not had sexual intercourse in months because he has not been able to have an erection. He says that it is so upsetting that they have basically stopped trying because it just makes both of them “depressed”. He has seen so many television commercials lately that he expects a cure in the form of a “little blue pill.” He is married, has 3 kids (age 1,3, and 6), works as a narcotics police officer, and competes in triathalons on the weekends. He takes no medications, rarely drinks alcohol, and has had no serious medical conditions. Physical examination is normal. The most appropriate next step in evaluating this patient’s erectile disorder is to ask him

  A. “Are you feeling unusually anxious lately?”
  B. “Are you sexually attracted to your wife?”
  C. “Do you have nocturnal or early morning erections?”
  D. “Do you love your wife?”
  E. “How often do you ride your bicycle?”
Explanation:

The correct answer is C. The main issue in evaluating impotence is distinguishing between psychological causes and organic causes. The presence of nocturnal or early morning erections basically eliminates the organic causes and leads to a diagnosis of psychological impotence. Up to 90% of erectile disorders are due to psychogenic factors. The most common psychological causes include anxiety and depression. Nocturnal penile tumescence, which occurs during REM sleep, can be assessed in a sleep lab or by a stamp test (wrapping stamps around the penis before bedtime and checking in the morning if the “ring” is broken at any of the perforated areas). If nocturnal erections are not present, the impotence is most likely due to an organic cause. The most likely causes are testicular failure, hyperprolactinemia, medications, alcohol, opioids, nicotine, trauma, priapism, diabetes, vascular disease, and neurologic diseases, such as diseases of the spinal cord, and loss of sensory input. Physical examination should include a detailed genital exam, evaluation for signs of feminization, neurologic, and vascular exams. “The little blue pill” (Viagra) is not indicated in a patient with psychogenic erectile disorders.

“Are you feeling unusually anxious lately?” (choice A) is a good question to follow the question about nocturnal erections (if he is having them).

“Are you sexually attracted to your wife?” (choice B) is a relevant question if the patient is having nocturnal erections and is not depressed or anxious. “Do you love your wife?” (choice D) is a question that may be asked if he is having nocturnal erections and is anxious and depressed. It may be a little blunt and can probably be asked in a more subtle manner.

“How often do you ride your bicycle?” (choice E) may be relevant because of neurologic and vascular compromise caused by the seat. However, it is not the most helpful question to distinguish between organic and psychologic causes.

 

A 27-year-old man comes to the emergency department because of right-sided scrotal pain and swelling which has worsened during the past 12 hours. He complains of severe pain, 10 out of 10 on the pain scale, which developed suddenly. The pain is radiating up to his right inguinal region. He denies any history of a similar problem in the past and reports no history of any genitourinary disease. His past medical and surgical histories are noncontributory. He takes no medications and has no known drug allergies. His social history reveals social alcohol use on the weekends and an occasional marijuana cigarette. He is sexually active with his girlfriend, has no other partners, and does not use protection. His temperature is 38.3 C (101.0 F), blood pressure is 150/80 mm Hg, pulse is 98/min, and respiratory rate is 22/min. Physical examination shows a soft, non-tender abdomen with normal active bowel sounds. His right testicle and epididymis are both enlarged and extremely tender. When the scrotum is gently elevated, the pain is mildly relieved. The rest of his physical exam is normal. Laboratory studies show a white blood cell count of 15,500/mm3 and a urinalysis positive for moderate leukocyte esterase and moderate nitrite. A scrotal ultrasound reveals enlargement of the right testicle and epididymis as well as increased blood flow to the right hemiscrotum. You make a diagnosis of acute epididymoorchitis. The most appropriate pharmacotherapy for this patient is

  A. ceftriaxone
  B. doxycycline
  C. doxycycline and ceftriaxone
  D. metronidazole
  E. metronidazole and ceftriaxone
Explanation:

The correct answer is C. Epididymoorchitis is a common condition seen in the emergency department. In sexually active men under the age of 35, a sexually transmitted disease is the most common etiology, specifically urethritis. These patients may present with epididymoorchitis without any history of symptoms of urethritis. In this population, urethritis is most commonly caused by N. gonorrhea and C. trachomatis. It is recommended that antibiotic treatment cover both organisms especially since gonococcal urethritis is associated with concomitant C. trachomatis infection in approximately 30-50% of cases. Treatment consists of a simple one time intramuscular injection of 250 mg of Rocephin to cover gonorrhea in addition to doxycycline 100 mg PO bid for 10 days to cover chlamydia.

Ceftriaxone(choice A) only is insufficient treatment due to common concomitant gonococcal and chlamydial infection. Ceftriaxone only covers a gonococcal infection.

Doxycycline(choice B) only is insufficient treatment as well. It only covers a chlamydial infection.

Metronidazole(choice D) only covers anaerobic organisms. These organisms do not commonly cause epididymoorchitis, therefore, metronidazole is a poor choice.

Metronidazole and ceftriaxone (choice E) in combination is a poor choice due to the lack of antibiotic coverage for chlamydia.

 

 

A 26-year-old man comes to the office because of a 3-day history of left-sided scrotal pain and swelling. He states that he is “very sexually active” and has had many sexual partners. He recently returned from a week-long Caribbean cruise, where he met “lots of other eligible partners.” His temperature is 38.2 C (100.8 F), blood pressure is 120/70 mm Hg, and pulse is 80/min. Examination shows unilateral intrascrotal tenderness. The scrotal skin is erythematous, warm, and there is a partial obliteration of the rugal folds. Testicular support makes the pain less intense. There is mucoid discharge present at the urethral opening. The most appropriate next step is to

  A. give him an immediate referral to a local urologist
  B. obtain a urethral culture and prescribe a 10-day course of oral ofloxacin
  C. order testicular ultrasonography
  D. send him to the emergency department for immediate surgical evaluation
  E. recommend scrotal elevation, ice packs, and nonsteroidal anti-inflammatory drug
Explanation:

The correct answer is B. This patient has epididymitis, most likely due to Chlamydia trachomatis. Epididymitis is the inflammation of the epididymis, which leads to unilateral intrascrotal pain, swelling, and fever. Testicular support usually relieves the pain to some extent. Asymptomatic urethritis can be associated with epididymitis due to C. trachomatis or N. gonorrhoeae. The discharge is mucoid in chlamydia and purulent in gonorrhoeae. Urinalysis may reveal pyuria. Urethral culture and urine PCR or LCR can confirm the diagnosis. The specific treatment of a C. trachomatis infection is azithromycin, doxycycline, or tetracycline, and the treatment for N. gonorrhoeae is a single dose of Ceftriaxone, given intramuscularly. A 10-day course of oral ofloxacin can treat both infections. The sexual partners should be treated to avoid reinfection.

A referral to a urologist (choice A) is unnecessary for a case of epididymitis or urethritis. A physical examination, urethral culture, and antibiotics are indicated and this can be done by a primary care physician and does not require a specialist.

A testicular ultrasound (choice C) is part of the initial evaluation of a testicular tumor, which often presents in this age group with a mass. A dull, aching pain may be present. This patient’s acute symptoms and urethral discharge make the diagnosis more consistent with an infectious process than a tumor. An ultrasound is also sometimes used to evaluate for testicular torsion.

Testicular torsion (choice D) is a surgical emergency, which is caused by twisting of the spermatic cord and vascular compromise. It is typical in young adults who present with acute, unilateral testicular pain and normal urinalysis results. Testicular support does not relieve the pain. If this is suspected, an immediate surgical evaluation is necessary.

In addition to antibiotic therapy, it is appropriate to recommend scrotal elevation, ice packs, and nonsteroidal anti-inflammatory drugs (choice E) to a patient with epididymitis. A urethral culture and antibiotics should be the initial steps before this treatment.

 

 

An 18-year-old man comes to the clinic with complaints of “scrotal discomfort” for the past several months. He also feels that there may be a mass around the testicle that he has felt for several years, but it never bothered him prior to this. He denies fever, dysuria, or urgency. He is not sexually active. His temperature is 37.2 C (99 F), blood pressure is 112/70 mm Hg, pulse is 64/min, and respirations are 14/min. His abdominal examination is unremarkable. The scrotum appears normal. However, on palpation, there is an extrascrotal mass, which feels somewhat like a “bag of worms”. Urinalysis is normal. A testicular ultrasound shows multiple dilated veins in the left hemiscrotum, which increase in size and in Doppler color flow with Valsalva maneuver. The left testicle is slightly smaller than the right. The most appropriate next step in management is to

  A. explain that no treatment is necessary at this time and a follow up is not necessary
  B. prescribe trimethoprim-sulfamethoxazole tablets, 1 by mouth daily for 1 week then reassess
  C. prescribe trimethoprim-sulfamethoxazole tablets, 1 by mouth twice daily for 1 month and then reassess
  D. refer him to a urologist for evaluation for a possible orchiectomy
  E. refer him to a urologist for an evaluation for surgical repair
Explanation:

The correct answer is E. This patient has the classic physical (“bag of worms”) and ultrasonographic findings, (dilated veins which distend on Valsalva) of a varicocele. A varicocele is an abnormal dilation of the spermatic veins within the scrotum. It can occur in up to 15% of adult and adolescent males. They can present with scrotal discomfort, but are often asymptomatic and the patient may just feel a mass. They also may be discovered in the workup of infertility, as varicoceles are a known contributing factor. The role in infertility is postulated to be due to several possible factors including increased temperature, increased hydrostatic pressure, and reflux of adrenal metabolites to name a few. The left-sided predominance of these lesions is thought to be because of the increased length of the left spermatic vein and its insertion at a right angle into the left renal vein. Adolescents with varicoceles are at a risk of losing testicular volume on the affected side as this patient is showing signs of. Surgical repair has been shown to decrease this growth retardation. For these reasons, no treatment or follow up (choice A) is not appropriate.

Trimethoprim-sulfamethoxazole (choices B and C) is commonly used in genitourinary tract infections. This patient has no signs of infection at this time.

An orchiectomy (choice D) would be performed for testicular malignancies or possibly in a torsed testicle that is necrotic. It is not typically performed for a varicocele.

 

 

A 54-year-old man presents to his primary care physician’s office over a concern regarding prostate cancer. The patient has no history of the disease, but his father died of prostate cancer at the age of 61 and the patient was told that he has an increased risk for developing the cancer. The patient reports that he has had digital rectal examinations each year, but that he would like to be “screened” for prostate cancer. He has no other medical history and takes only a low-dose aspirin daily. He denies smoking and illicit substance abuse and admits to drinking alcohol socially.The most appropriate response to this patient is:

  A. “Normal digital rectal examinations exclude any likelihood of prostate cancer.”
  B. “PSA blood tests are available.”
  C. “PSA blood tests are available but only for patients with known cancer.”
  D. “PSA urine tests are available for all men over the age of 50.”
  E. “There is no effective screening test for prostate cancer.”
Explanation:

The correct answer is B. Men have between a 15-20% lifetime risk of developing prostate cancer. The PSA test was introduced in the late 1980s and quantifies a glycoprotein produced by the prostate that spills over into the blood. Although current guidelines differ by society, clinical practice and standard of care is to test high-risk men over the age of 40 for PSA levels. Any level greater than 4.0 ng/mL requires a prostate biopsy. The positive and negative predictive values of the test vary tremendously with patient population, but roughly 30% of patients with elevated PSA levels will have prostate cancer. These tests are also used to follow therapy in patients with known prostate cancer (choice C).

Although digital rectal examinations (choice A) are capable of detecting enlarged prostates and nodules, they do not effectively exclude small tumors, which fail to distort the gland or that present in areas of the gland which are not peri-urethral.

PSA tests are for detection of PSA in blood, not urine (choice D). There is no PSA present in ejaculate or in urine although ejaculation does transiently increase serum PSA levels for up to 48 hours.

Although the PSA test has variable sensitivity and specificity, it is a fairly robust test when used to screen patients at high risk or with enlarged prostate glands on physical examination. Therefore, it is an effective screening (choice E) test, but the efficacy depends very much on the population being tested and specific characteristics of the patient such as the presence or absence of benign hypertrophy.

 

 

A 51-year old married man comes to the office complaining of blood in his semen. He states that approximately 2 weeks prior to presentation he noticed bloody ejaculate. There were 2 episodes within 3 days of each other. Since the last episode he has had normal ejaculations on multiple occasions. There is no associated pain, penile discharge, erectile dysfunction, abdominal pain, or history of trauma. His medical history is significant for diet-controlled diabetes and eczema. There is no family history of prostate cancer. Physical examination reveals no abnormalities of the penis or scrotum. On digital rectal examination, his prostate is smooth, non-tender, firm, normal in size, and without nodule. Serum prostate specific antigen (PSA) is 1.4 ng/mL. Urinalysis and urine cytology are both negative. The next best step in management is

  A. observation and reassurance
  B. obtain a semen analysis
  C. order a transrectal ultrasound
  D. perform cystoscopy or cystourethroscopy
  E. prescribe ciprofloxacin for 7 days
  F. repeat PSA
  G. schedule a prostate biopsy
Explanation:

The correct answer is A. Hematospermia, the presence of blood in the seminal fluid, is usually the result of nonspecific inflammation of the urethra, prostate, or seminal vesicles. It is almost always self-limiting and resolves within several weeks. Occasionally, hematospermia may be associated with infection, particularly tuberculosis, cytomegalovirus, and schistosomiasis, but rarely is it associated with malignancy. All patients with this complaint should undergo careful physical exam to exclude hypertension which may cause hematospermia, a rectal exam, and a PSA test to exclude prostatic carcinoma, and a urine cytology to rule out the possibility of transitional cell carcinoma of the prostate. This patient has had a complete work-up for initial presentation of hematospermia. There is no need to do anything but reassure the patient that his condition is almost always self-limiting.

Semen analysis (choice B) evaluates semen for volume and sperm concentration, quantity, motility, and morphology. Its utility is in the evaluation of infertility.

Transrectal ultrasonography (choice C) can evaluate the prostate, seminal vesicles, or ejaculatory ducts. It may be utilized for persistent hematospermia. It does not play a role in evaluation of a single episode of hematospermia.

Cystoscopy, or cystourethroscopy (choice D), allows for visualization of the pendulous, bulbar and prostatic urethra and bladder, and is not indicated unless hematospermia persists and there is concern for the underlying pathology.

Antibiotics (choice E) can be utilized in the treatment of prostatitis. There is no evidence that reveals utility of such agents in the current clinical scenario. When treating prostatitis, antibiotics should be used for at least 21 days. Patients with prostatitis normally have a soft, boggy, and tender prostate on rectal exam and have associated urinary complaints (dysuria and frequency).

Prostate specific antigen (PSA) (choice F) is a useful screening tool for prostate cancer. In general, a normal PSA is any value less than 4. Some urologists also believe in age-adjusted PSA with a normal value being slightly lower for younger patients. A repeat exam is not necessary in light of this patient’s normal level.

Prostate biopsy (choice G) is an invasive procedure that is performed in patients who have a clinical suspicion of prostate cancer (i.e., abnormal digital rectal exam or elevated PSA). This patient has neither, and subjecting him to an unnecessary invasive test is not indicated.